It would mean the world to me if some one can answer both questions 1 and 2 please help

It Would Mean The World To Me If Some One Can Answer Both Questions 1 And 2 Please Help

Answers

Answer 1

Answer:

the answer to both are 1. 3.77 2.19.94


Related Questions

Question is in photo.
Thanks

Answers

no, the dots make no straight lines

Marissa went to the store with $20 and returned home with $5.36. If she bought only 3 gallons of ice cream, what was the cost of each gallon of ice cream?

Answers

Answer: $4.88

Explanation: $20.00-$5.36= $14.64

$14.64÷3= $4.88

solve: 31 - 8(y+5) =7

Answers

Answer:

y = -2

General Formulas and Concepts:

Pre-Algebra

Order of Operations: BPEMDAS

Brackets Parenthesis Exponents Multiplication Division Addition Subtraction Left to Right  

Equality Properties

Multiplication Property of Equality Division Property of Equality Addition Property of Equality Subtraction Property of Equality

Step-by-step explanation:

Step 1: Define

Identify

31 - 8(y + 5) = 7

Step 2: Solve for y

[Subtraction Property of Equality] Subtract 31 on both sides:                       -8(y + 5) = -24[Division Property of Equality] Divide -8 on both sides:                                y + 5 = 3[Subtraction Property of Equality] Subtract 5 on both sides:                        y = -2
31-8y-40=7
-8y-9=7
-8y=7+9
Y=16/-8

Final answer is y= -2

The product of two integers is 84. If one
integer is (-12), find the other.​

Answers

Answer:

-7

Step-by-step explanation:

-12x = 84

/-12     /-12

   x = -7

Answer:

The other integer is - 7.

Step-by-step explanation:

Let the 2 integers be x and y

Given:

  one of the integer is -12, Let x = - 12

  Product of two integers is 84,

that is,

       x × y = 84

     - 12 × y = 84                        [ given one of the integer is -12 ]

       [tex]\frac{-12 \times y }{-12} = \frac{84 }{-12}[/tex]                      [ divide both sides by -12 ]

        [tex]y = - 7[/tex]

How do you calculate angles on a straight line??

Answers

Answer:

Step-by-step explanation:

Straight line is an angle whose vertex point(O) has a value of 180. The arms (OA & OB) of the straight line angle lies opposite to each other from the vertex point

Sum of all the angles in a straight line  = 180

From this you can find the value of unknown angle, if other angles are given.

∠AOC + ∠BOC = 180

Answer:

Answer:

x + y =180 (degrees)

Can someone please figure this out thank you

Answers

Using the tangent ratio below:

[tex] \large \boxed{tan A = \frac{opposite}{adjacent} }[/tex]

Therefore, the tangent ratio for a right triangle is:

[tex] \large{tanA = \frac{8}{12} }[/tex]

For this part, use the arctan to find the measure of A.

[tex] \large{arctan( \frac{8}{12} ) = A} \\ \large{arctan( \frac{8}{12} ) = 33.69 \degree}[/tex]

Therefore measure of A is 33.69, round to the nearest tenth as we get 33.7 degrees.

Answer

A = 33.7 degrees

Furthermore, the question mentions the nearest tenth, that means the b and e choice should be cleared out.

Let me know if you have any doubts regarding the trigonometric ratio.

12-2²·2=?

Brainliest

Answers

Answer:

it would be 4

12-4×2=12-8=4

Step-by-step explanation:

hope it helps you

[tex]\longrightarrow{\green{ 4 }}[/tex] 

[tex]\large\mathfrak{{\pmb{\underline{\red{Step-by-step\:explanation}}{\red{:}}}}}[/tex]

➺ [tex] \: 12 - {2}^{2} .2[/tex]

➺ [tex] \: 12 - (2 \times 2 \times 2)[/tex]

➺ [tex] \: 12 - 8[/tex]

➺ [tex] \: 4[/tex]

[tex]\large\mathfrak{{\pmb{\underline{\orange{Mystique35 }}{\orange{❦}}}}}[/tex]

The volume of 10 drops of liquid is 0.1 fluid ounces.
What is the volume of 10,000 drops

Answers

Answer:

10,000 drops would be 1 fluid ounce

Step-by-step explanation:

6th grade math help me pleaseeee

Answers

Answer:

The answer is $38115

Step-by-step explanation:

Because I have the smarts


Need help now I have 3 min asapppppp

Answers

Answer:

Step-by-step explanation:

a:0

B:1

C:-1

In figure, if PQ ⊥ PS, PQ||SR, ∠SQR = 2S° and ∠QRT = 65°, then find the values of x and y.​

Answers

Step-by-step explanation:

YAY!!!!!!!!!!!!!!!!!!!

What of the following is the BEST statement about dependent and independent variables? Group of answer choices An experiment can have more than one of either kind of variable; you just have to keep your data straight. An independent variable is a factor that changes because of procedures conducted on the dependent variable. A dependent variable is a factor that changes because of procedures conducted on the independent variable. An experiment can have several independent variables but only one dependent variable.

Answers

Answer:

A dependent variable is a factor that changes because of procedures conducted on the independent variable

Step-by-step explanation:

Using a cause and effect analogy, the independent variable is the causal variable. The independent variable or variables are the variables on which what is being measured, has no effect, and it is the variable the investigator manipulates to find the effect on the dependent variable.

The dependent variable are the variables that shows he effect of the changes made and manipulations to the independent variable. The dependent variable is therefore, the effect variable

Therefore, the correct option is ' A dependent variable is a factor that changes because of procedures conducted on the independent variable'

line RP and line RQ are tangent to point G at P and Q. if the measurement of angle PRG=35 degrees, find the measurement of angle PGR.​

Answers

Answer:

55°

Step-by-step explanation:

By theorem of specific circle, the radius is perpendicular to the tangential lineSummation of all internal angles equal to 180°

Thus from the atrachment, the angle PGR is 180°-35°-90°

= 55°

The measures of the angles of a triangle are shown in the figure below. Solve for x.

Answers

Answer and Step-by-step explanation:

All triangles have a sum of their angles to be 180 degrees. So, to find x, we need to subtract the two given sides from 180 degrees.

180 - 38 - 83 = 59

x = 59 degrees.

#teamtrees #PAW (Plant And Water)

Answer: 59

Step by step explanation:

Compute the missing data in the table for the following exponential function f (x) = (one-fourth) Superscript x.

Answers

Answer:

[tex] F(4) = \frac {1}{256} [/tex]

Step-by-step explanation:

Given the following mathematical function;

[tex] F(x) = \frac {1}{4^{x}} [/tex]

When x = 1

[tex] F(1) = \frac {1}{4^{1}} [/tex]

[tex] F(1) = \frac {1}{4} [/tex]

When x = 2

[tex] F(2) = \frac {1}{4^{2}} [/tex]

[tex] F(2) = \frac {1}{4*4} [/tex]

[tex] F(2) = \frac {1}{16} [/tex]

When x = 3

[tex] F(3) = \frac {1}{4^{3}} [/tex]

[tex] F(3) = \frac {1}{4*4*4} [/tex]

[tex] F(3) = \frac {1}{4*4*4} [/tex]

When x = 4

[tex] F(4) = \frac {1}{4^{4}} [/tex]

[tex] F(4) = \frac {1}{4*4*4*4} [/tex]

[tex] F(4) = \frac {1}{256} [/tex]

When x = 5

[tex] F(5) = \frac {1}{4^{5}} [/tex]

[tex] F(5) = \frac {1}{4*4*4*4*4} [/tex]

[tex] F(5) = \frac {1}{1024} [/tex]

The answer to this.

Answers

Answer:

The two column proof is presented as follows;

Step [tex]{}[/tex]     Statement               Reason

1 [tex]{}[/tex]            [tex]\overline {AC}[/tex] ≅ [tex]\overline {BD}[/tex]                Given

[tex]{}[/tex]              ∠CAB ≅ ∠DBA  

2  [tex]{}[/tex]          [tex]\overline {AB}[/tex] ≅ [tex]\overline {AB}[/tex]                Reflexive property

3 [tex]{}[/tex]          ΔABC ≅ ΔBAD        SAS rule of congruency

Step-by-step explanation:

Given that we have;

Segment [tex]\overline {AC}[/tex] of ΔABC being congruent to (≅) segment  [tex]\overline {BD}[/tex] on ΔBAD and angle ∠CAB on ΔABC is congruent to angle ∠DBA on ΔBAD, and also that the two triangles share a common side, which is segment [tex]\overline {AB}[/tex], we have;

Segment [tex]\overline {AB}[/tex] is congruent to itself by reflexive property, therefore;

Two sides and an included angle on ΔABC are congruent to the corresponding two sides and an included angle on ΔBAD, which by Side-Angle-Side, SAS, rule of congruency, ΔABC is congruent to ΔBAD

Can someone please please please help me and show working I am stuck.​

Answers

Answer:

i think the c answer is 5

Step-by-step explanation:

Franklin and Glennys are selling cases of chocolate bars for a school fundraiser. Franklin sells 5 cases and Glennys sells 8 cases. There are x bars of chocolate in each case. How many chocolate bars does Franklin sells?

Answers

Answer:

Franklin sold 5x bars.

Step-by-step explanation:

Franklin Sells 5x bars. I assume the question is asking for an expression with the usage of X instead of an actual value or when that is attainable. If there's any more information that you can provide, let me know and I'll redo the problem.

if A={2,3,5,7,9,10} B={2,10,12,13} then a-b is​

Answers

A = {2,3,5,7,9,10}

B = {2,10,12,13}

A-B = ?

Now,

A-B = {2,3,5,7,9,10} - {2,10,12,13}

= {3,5,7,9}

I hope you understand...

Mark me as brainliest...

if there are 3 odd numbers and 2 even numbers what is the probability

Answers

There is 5 total. Umbers and 3 of them are odd.

The probability of getting odd is the number of odd over total numbers:

Probability of odd = 3/5

Answer:

P ( odd) = 3 / 5

Step-by-step explanation:

Total number = 3 + 2 = 5

Odd number = 3

The probability of odd number is = odd number / total number

P ( odd )= 3 / 5

In ΔBCD, the measure of ∠D=90°, BD = 13, DC = 84, and CB = 85. What is the value of the cosine of ∠C to the nearest hundredth?

Answers

Answer: 0.99

Step-by-step explanation: sin 90 Divided by 85 = sin x divided by 13

                                                   sin x =13x sin 90 divided by 85

                                                        sin x = 13divided by 85

                                                            sin x = .152941176

                                                arc sine of  .152941176=  8.79741071 degrees

                                        cos 8.79741071 = .988235294

                                                            .99

                                                 

please help me with this real quick.​

Answers

the answer is the last option

Expand. Your answer should be a polynomial in standard form.
8r^2(r^2-2)=

Answers

Answer:

8r^4 - 16r^2

2Step-by-step explanation:

8r^2(r^2-2)

Multiply everything inside the brackets by the term outside of it.

8r^4 - 16r^2

The plans for a new aquarium call for 2 hallways of exhibits leading out of a circular main room as shown. What is the value of x? Give reason

Answers

Answer:

[tex]x =23^o[/tex]

Step-by-step explanation:

Given

See attachment

Required

Find x

To solve for x, we make use of:

[tex]5x + 65= 180[/tex] --- angle on a straight line

[tex]5x = 180-65[/tex]

[tex]5x = 115[/tex]

Solve for x

[tex]x = 115/5[/tex]

[tex]x =23^o[/tex]

Trigonometry help me

Answers

Answer:

[tex]\theta = \frac{\pi}{6}[/tex]

Step-by-step explanation:

[tex]tan^ 2 \theta - ( \sqrt 3 + \frac{1}{\sqrt3}}) tan \theta + 1 = 0\\\\tan \theta - ( \sqrt 3 + \frac{1}{\sqrt3}}) +\frac{1}{ tan \theta } = 0\\\\[/tex]          [tex][ \ divide \ by \ tan \theta \ on \ both \ sides \ ][/tex]

[tex]tan\theta + \frac{1}{ tan \theta }- ( \sqrt 3 + \frac{1}{\sqrt3}}) = 0\\\\\frac{tan^2 \theta + 1}{ tan \theta } - ( \sqrt 3 + \frac{1}{\sqrt3}}) = 0\\\\\frac{sec ^2 \theta}{ \frac{sin \theta }{cos \theta}} - ( \sqrt 3 + \frac{1}{\sqrt3}}) = 0[/tex]              [tex][ \tan ^ 2\theta + 1 = sec ^2 \theta \ , \ tan \theta = \frac{sin \theta }{cos \theta } \ ][/tex]

[tex]\frac{sec^2 \theta }{sin \theta \times sec \theta } - ( \sqrt 3 + \frac{1}{\sqrt3}}) = 0\\\\[/tex]                  [tex][\ \frac{sin \theta }{cos \theta } = sin \theta \times sec \theta \ ][/tex]

[tex]\frac{sec \theta }{sin \theta } - ( \sqrt 3 + \frac{1}{\sqrt3}}) = 0\\\\[/tex]

[tex]sec \theta \ cosec \theta - ( \sqrt 3 + \frac{1}{\sqrt3}}) = 0\\\\[/tex]              [tex][ \ \frac{1}{sin \theta } = cosec \theta \ , \ \frac{ sec \theta }{sin \theta } = sec \theta cosec \theta \ ][/tex]

[tex]sec \theta \ cosec \theta - \sqrt 3 - \frac{1}{\sqrt3}} = 0\\\\\frac{\sqrt 3\ sec \theta \ cosec \theta - 3 - 1}{\sqrt3} = 0\\\\\sqrt 3 sec \theta cosec \theta - 4 = 0\\\\[/tex]              

[tex]\sqrt3 \frac{1}{cos \theta } \frac{1}{sin \theta } - 4 = 0\\\\\frac{\sqrt3 - 4sin \theta cos \theta} { sin \theta cos \theta } = 0[/tex]                      

[tex]\sqrt 3 - 2sin 2\theta = 0[/tex]                                  [tex][ \ sin 2 \theta = 2 sin \theta cos \theta \ ][/tex]

[tex]2sin 2 \theta = \sqrt3\\\\sin 2 \theta = \frac{\sqrt3 }{2} \\\\2 \theta = sin^{-1} (\frac{\sqrt3}{2})\\\\2 \theta = 60^{ \circ} = \frac{ \pi}{3}\\\\\theta = \frac{\pi} {6}[/tex]

Which graph shows the solution to the system of linear inequalities?

y > Two-thirdsx + 3

y ≤ Negative one-thirdx + 2

Answers

Answer:

b

Step-by-step explanation:

We want to get the graph for the given system of linear inequalities. The graph can be seen at the end of the answer.

How to graph a system of inequalities?

First, we have the system:

y > (2/3)*x + 3

y ≤ -(1/3)*x + 2

Let's analyze each one individually.

In the first one, you can see that y is larger than the line, so we need to shade the region above the line, and the line must be a dashed line, meaning that the points on the line are not solutions.

In the second one, y is smaller than or equal to the line, so we need to shade the region below the line, and this time the line must be a solid line because the points on the line are solutions.

The graph of the system of inequalities can be seen below.

If you want to learn more about inequalities, you can read:

https://brainly.com/question/11234618

In a three-digit number, the hundreds digit is one half of the tens digit. The tens digit is
twice the one's digit. If the sum of the digits is eight, find the number.

Answers

Answer:

242

Step-by-step explanation:

Rule out anything above five from being the ones number as it wouldn't be possible to be doubled for the tens.

After that it is just trial and error putting 1,2,3,4 in the tens and following the word equation.

121 = doesn't equal 8

363= doesn't

484 = doesn't

thats leaves 242 which follows the pattern and equals 8

A sample of 12 measurements has a mean of 16.5, and a sample of 15 measurements has a mean of 18.6. Find the mean of all 27 measurements

Answers

Answer: [tex]17.67[/tex]

Step-by-step explanation:

Given

Sample  of 12 measurements has a mean of 16.5 and

a sample of 15 measurements has a mean of 18.6

Take [tex]\bar{x_1},n_1[/tex] be the mean and no of measurements

and [tex]\bar{x_2},n_2[/tex] be the mean and no of measurements in second case

[tex]\therefore \bar{x_1}=\dfrac{\sum a_1}{n_1}\\\\\Rightarrow \sum a_1=\bar{x_1}\times n_1\\\\\Rightarrow \sum a_1=198\quad \ldots(1)[/tex]

Similarly,

[tex]\therefore \bar{x_2}=\dfrac{\sum a_2}{n_2}\\\\\Rightarrow \sum a_2=\bar{x_2}\times n_2\\\\\Rightarrow \sum a_2=279\quad \ldots(2)[/tex]

Mean of 27 measurements

[tex]\Rightarrow \bar{x_3}=\dfrac{\sum a_1+\sum a_2}{n_1+n_2}\\\\\Rightarrow \bar{x_3}=\dfrac{198+279}{12+15}\\\\\Rightarrow \bar{x_3}=\dfrac{477}{27}\\\\\Rightarrow \bar{x_3}=17.67[/tex]

if you apply the changes below to the linear parent, f(X)=x, what is the equation of the new function vertically stretch by multiplying by 3. flip over the x-axis

Answers

Answer:

f(x) = -3x

Step-by-step explanation:

f(x) = -3x

How many sides do 1 decagon, 3 heptagons, 3 nonagons, and 3 hexagons have in all?

Answers

Answer:

76

Step-by-step explanation:

decagon = 10 sides

3 heptagons = 3 × 7 = 21 sides

3 nonagons = 3 * 9 = 27 sides

3 hexagons = 3 * 6 = 18 sides

10 + 21 + 27 + 18 = 76 sides in total

hope this helps please like and mark as brainliest

Other Questions
how community can benefit in living in a healthy environment [tex]huge\color{red}Testing[/tex] Dcrivez votre ami (e) (environ 40 mots) Help me please the numbers I am using is 1,2,2,3,4,5,6,7,8,9 Why is Jeannette more comfortable with Dinita than she is with white children? If you apply these changes to the linear parent function, f(x) = x, what is theequation of the new function? Vertically compress by a factor of 7. Shift up 9 units.O A. DY) = 7x+9O B. g(x) = = x+9O C. () = 7(x-9)O D. g(x) = + (x+9) Baseball Corporation is preparing its cash budget for January. The budgeted beginning cash balance is $18,600. Budgeted cash receipts total $186,000 and budgeted cash disbursements total $189,200. The desired ending cash balance is $30,600. To attain its desired ending cash balance for January, the company should borrow: Which of these is the most likely outcome of a financial crisis?A. Lower unemploymentB. More businesses failingC. Fewer foreclosuresD. Greater company profits For the function f(x)=7x^38x+2x^2, Step 1 of 2 : Find the slope of the tangent line at x=1. 2) If the question asked why the boy went to thepark, and your answer is 2:00 p.m., does youranswer make sense? Choose an equation of a line through the point (-4, 1) perpendicular to y=12x+3. Which of these was a factor that pulled immigrants to the United StatesA. Dirty, crowded cities B. Job opportunities C. Lack of foodD. Political unrest What is the complement that is given from the angle? Solve for x 11^x = 11^4* 11^5 Find the height of this triangle. Economists look at any situation in terms of its component parts: the people making decisions, the environment in which they're making those decisions, and the goods or services being exchanged. For example, think about doing your laundry. Which of the following laundry-related items are nondurable goods?a. A washing machine b. Having a pair of pants dry-cleaned c. A T-shirt d. A clothes dryer Have you ever had to work so hard at something you almost gave up? What was it?And did you give up? quien quiere ser mi novia what is meaning of marshmello? Anita can exercise for 20 minutes on her lunch break and 30 minutes after work on her 5 work days. She exercises 35 minutes on the weekend. How many hours per week does Anita exercise? 4 hours per week 4 hours per week 4 hours per week 5 hours per week